Reproductive Pharmacology

अब Quizwiz के साथ अपने होमवर्क और परीक्षाओं को एस करें!

A 46-year-old female presents with a primary complaint of irregular menstrual bleeding that has persisted for the last several months. Her medical history consists of 2 full-term pregnancies and invasive ductal carcinoma of the breast that was successfully treated one year ago. After an appropriate evaluation, she is given a diagnosis of type 1 endometrioid adenocarcinoma. Which of the following factors most likely increased this patient's risk for developing this cancer? 1. Administration of raloxifene in the treatment of her previous breast cancer 2. The patient is 46-years-old and has yet to enter menopause 3. Combined oral contraceptive use from the age of 32 to 42 4. The patient's two previous pregnancies 5. Tamoxifen regimen used to treat her previous breast cancer

5. Tamoxifen regimen used to treat her previous breast cancer

What is the last step in estrogen synthesis?

Aromatization ***(wha does aromatase do?)

A 60-year-old Caucasian male comes to your office with a 2-month history of back pain that is not responsive to over-the-counter pain relievers. The pain is worse at night and interferes with his sleep. Lumbar vertebrae are tender to percussion, and the prostate is enlarged and firm. After a thorough evaluation, you decide to proceed with leuprolide therapy. Which of the following changes in testosterone (T) and dihydrotestosterone (DHT) levels are most likely in this patient after initiation of the therapy? A. Steady decrease in both T and DHT levels B. First concordant increase, then concordant decrease in T and DHT levels C. Discordant decrease in DHT level D. First discordant increase, then discordant decrease in DHT level E. No change in T and DHT levels

B. First concordant increase, then concordant decrease in T and DHT levels Pulsatile release of gonadotropin-releasing hormone (GnRH) from the hypothalamus is the natural state of human functioning, causing release of gonadotrophins from the pituitary that in turn stimulates release of testosterone in a man. Testosterone is converted to DHT in target tissues by the enzyme 5-alpha reductase. Alternatively, if GnRH levels are constantly elevated, rather than pulsed, the secretion of luteinizing hormone (LH) and follicle-stimulating hormone (FSH) from the pituitary will be suppressed. A number of GnRH analogs, such as leuprolide, have been generated by amino acid substitutions-alterations that allow longer half lives and increased activity. Leuprolide is a long-acting GnRH analog that causes continuous GnRH activity. The result is ultimately a suppression of the pituitary-gonadal axis for the duration of treatment, although there is a brief period of initial stimulation, sometimes called a "start up flare." An increase in gonadotrophin levels during the initial flare-up period causes an increase in both testosterone and DHT levels (a concordant increase). Soon the start up flare burns out, and both testosterone and DHT are typically suppressed to castrate levels. These very low levels of androgens are useful for the treatment of androgen-dependent cancers, such as prostate cancer. (Choice A) With the use of a GnRH analog, androgen levels are suppressed after a transient increase, so the "decrease only" option here cannot be correct. Suppression of testosterone and DHT without an initial flare is seen with the use of GnRH antagonists. (Choices C and ID) Leuprolide has no effect on 5-alpha reductase activity. Therefore, the changes in testosterone and DHT levels are always concordant following leuprolide administration. Finasteride is a 5-alpha-reductase inhibitor used for the treatment of benign prostatic hypertrophy. It prevents the conversion of testosterone to DHT, causing a discordant decrease in DHT levels. (Choice E) The purpose of administering a GnRH is to change androgen levels (testosterone and DHT levels). so "no change" cannot possibly be correct. Educational Objective: Leuprolide is a GnRH agonist that causes first a transient increase, then a decrease in both testosterone and DHT levels. Finasteride causes a discordant decrease in DHT level

What are the two most common adverse effects of estrogen therapy?

Nausea and Breast tenderness

Adverse effects of Estrogen Therapy

Nausea,vomiting Breast Tenderness Increased risk of thrombotic events Myocardial infarction Breast and Endometrial Cancer

Mifepristone

*Also known as RU-486 *Typically used with Misoprostol to bring about an abortion * Smaller doses can be used as an emergency contraceptive* *Common side effects = abdominal pain, heavy bleeding, incomplete abortion *Works by blocking the effects of progesterone needed to maintain a pregnancy *Used up to 63 days of pregnancy (9 weeks) *Higher effectiveness when used with misoprostol vs when used alone

Leuprolide

*GnRH analog *Action depends on mode of administration *GnRH antagonist when infusion continuously *GnRH agonist when infused in a pulsatile manner (natural way the body releases GnRH) *Infertility (pulsatile) *Prostate cancer (continuous

Flutamide

*Nonsteroidal competitive inhibitor at androgen receptors *Used in the treatment of prostate carcinoma * *

Clomiphene

*Partial estrogen receptor agonist *Acts specifically in hypothalamus which antagonizes normal negative feedback inhibition (hypothalamus thinks that there is no estrogen) *Results in an ↑ release of LH and FSH from pituitary => stimulates ovulation *Useful for the treatment of infertility associated with anovulatory cycles

Combined Contraceptives

*Products containing a combination of an estrogen and a progestin are the most common type of oral contraceptives. * * * * *

Danazol

*Synthetic androgen that acts as partial agonist at androgen receptors *hypoestrogenic, hyperandrogenic effects * prevents ovulation by suppressing the increase of FSH and LH *inhibits ovarian steroidogenesis

A 24-year-old G0 woman presents to the ED of a local hospital stating that she was sexually assaulted. The patient is alone, and she is sobbing and is clearly anxious. She states that her last menstrual period was 3.5 weeks ago and asks the physician to please help her prevent a possible pregnancy from the assault. Physical examination is significant for bruised external labia and a .5-cm laceration in the vaginal vault. There is no vaginal discharge or cervical motion tenderness. After the appropriate psychological support is provided and a social worker is consulted, she is given one dose of 600 mg of mifepristone. Mifepristone contains which of the following? (A) Estrogen and progestin (B) High-dose estrogen only (C) Progestin only (D) Progesterone antagonist (E) Prostaglandin

D The correct answer is D. Mifepristone is an antiprogestin that has been shown to be 100% effective at preventing pregnancy when given within 72 hours of intercourse at a one-time dose of 600 mg.

MOA of Clomiphene

Clomifene inhibits estrogen receptors in the hypothalamus, inhibiting negative feedback of estrogen on gonadotropin release, leading to up-regulation of the hypothalamic-pituitary-gonadal axis => increased FSH and LH release from the pituitary

Leuprolide is a GnRH ___________ (agonist/antagonist) when infusion continuously; GnRH _________ (agonist/antagonist) when infused in a pulsatile manner

Continuously: GnRH antagonist Pulsatile: GnRH agonist GnRH is naturally released in a pulsatile manner, so when Leuprolide mimics this, it acts as an agonist

A 33-year-old female visits to your office and requests a simple and reliable method of contraception. Her past medical history is insignificant, she does not take any medications other than a daily multivitamin. Which of the following factors would most affect your decision to prescribe oral contraceptives to this patient? A. Diet B. Physical activity level C. Smoking status D. Parity E. Glucose intolerance

Correct Answer: C. Smoking status Explanation: Oral contraceptive pills are commonly used for female contraception. They typically contain a combination of estrogen and progestin, and are taken in cycles interrupted by menstrual bleeding. The estrogen in oral contraceptives prevents pregnancy by suppressing the midcycle gonadotropin surge, thereby inhibiting ovulation. Progesterone is added to counteract the increased risk of endometrial cancer associated with unopposed effect of estrogen. Side effects of oral contraceptives may include: breakthrough menstrual bleeding breast tenderness, and weight gain. Additionally, there is a risk of more serious, though rare, events, such as deep vein thrombosis pulmonary embolism ischemic stroke, and myocardial infarction. The risk of cardiovascular events due to oral contraceptive pills is increased in smokers and patients over the age of 35. In particular, individuals who smoke more than a pack of cigarettes per day have a much higher incidence of cardiovascular events. (Choices A and B) Use of oral contraceptive pills may cause weight gain. However there is no relation of weight gain to the more serious potential health risks associated with oral contraceptives. (Choice D) The risks of oral contraceptive pills are similar in females regardless of past pregnancy or childbearing status. (Choice E) Oral contraceptive pills can induce insulin resistance and cause glucose intolerance. However the risk of glucose intolerance was most significant with the first generation of high-dose estrogen oral contraceptives.

A 72-year-old male visits your office with complaints of increased frequency of urination and a painful mass in the perineal region. After thorough investigation, he is found to have metastatic prostate cancer and is started on flutamide. He reports significant relief of his bone pain soon after initiation of the therapy. The primary tumor decreases in size. Which of the following is the best explanation for the changes observed in this patient? A. Decreased Leydig cell stimulation B. Decreased Leydig cell androgen synthesis C. Decreased peripheral androgen aromatization D. Decreased peripheral androgen conversion E. Impaired ligand-receptor interaction

Correct Answer: E. Impaired ligand-receptor interaction Explanation: Flutamide is a non-steroid antiandrogen used for the treatment of prostate cancer. It competes with testosterone and DHT for their receptors in the target cells. Prostate cancer is testosterone-dependent and is treated with surgical and pharmacological androgen ablation. Prevention of androgen-receptor binding blocks the stimulatory effect of androgens on the primary tumor and metastases and leads to a reduction in their size. If flutamide is used as monotherapy, it causes a gradual increase in blood testosterone levels due to negative feedback mechanism. This effect is reduced by the simultaneous administration of GnRH agonists. Side effects of flutamide include those found in androgen depletion (hot flashes, gynecomastia, and impotence). (Choice A) Gonadotropin-releasing hormone (GnRH) agonists (leuprolide, goserelin, nafarelin and histrelin) bind to GnRH receptors in the anterior pituitary and inhibit synthesis of LH and FSHI if administered continuously. Decreased amount of LH leads to decreased Leydig cell stimulation and diminished testosterone synthesis. (Choice B) Ketoconazole is a weak anti androgen that decreases synthesis of steroid hormones in gonads and adrenals. Flutamide does not affect testosterone production by Leydig cells. (Choice C) Decreased peripheral androgen aromatization refers to the mechanism of anastrozole, a nonsteroidal aromatase inhibitor which blocks estrogen production selectively.. (Choice D) Finasteride decreases peripheral conversion of testosterone into dihydrotestosterone by inhibiting the 5-α- reductase. It is used for treatment of BPH and male baldness.

Mifepristone is used for: a. Treatment of premature labor b. Eclampsia c. Closure of patent ductus arteriosus d. Early abortion e. Asthma

D 1. Mifepristone is a synthetic steroid compound used as to induce abortion 2. It is a *progesterone receptor antagonist* used as an abortifacient in the first months of pregnancy, and in *smaller doses as an emergency contraceptive*. 3. It is used in the termination of pregnancies* less than sixty-four days from conception*. A typical dose is 200 mg orally, followed after 48 hours by 800 μg misoprostol, a prostaglandin analogue, given orally or vaginally. Within four hours of the second dose, it is between 92% and 99% effective (depending on the trial data examined), while as a standalone agent it is approximately 80% effective.

A 72-year-old male with a long history of benign prostatic hypertrophy is started in on an alpha-1 blocking medication that is specific for the alpha-1A and 1D receptor subtype. The medication most likely started is which of the following? A. Alfuzosin B. Prazosin C. Indoramin D. Tamsulosin E. Terazosin

D. Tamsulosin

A 75-year-old male is diagnosed with advanced metastatic prostate cancer. After further evaluation and staging, the patient is started on flutamide therapy. Addition of which of the following medications to this patient's medication regimen would be of greatest benefit in the treatment of this patient's condition? 1. Leuprolide 2. Anastrozole 3. Clomiphene 4. Tamoxifen 5. Cyproterone

1. Leuprolide Flutamide is a non-steroidal antiandrogen that acts as a competitive inhibitor at testosterone receptors. It is used in combination with a GnRH agonist, such as leuprolide, in the treatment of advanced prostate cancer. The inhibitory effect of flutamide on the tumor's testosterone receptors is beneficial to halt the progression of prostate cancer. However, there is also an unwanted increase in systemic testosterone levels seen with flutamide monotherapy due to reduced negative feedback at the level of the pituitary. The addition of a leuprolide counters this unwanted increase in testosterone levels by decreasing the release of LH and FSH from the anterior pituitary, thereby reducing androgen production. Combination flutamide/leuprolide therapy has side effects associated with reduced androgen levels, including hot flashes, gynecomastia, and impotence.

A 30-year-old woman is being treated with leuprolide for estrogen-dependent breast cancer. After 2 months of continuous treatment, which of the following is the most likely effect on the secretion of luteinizing hormone and follicle-stimulating hormone from the pituitary gland? LH FSH A) ↓ ↓ B) ↓ - C) ↑ ↑ D) ↑ - E) - ↓

A) ↓ ↓ Continuously: GnRH antagonist

A 37 yo woman G2P2 comes to the physician requesting contraceptive advice. she recently became sexually active with a new partner. she has no history of serious illness and takes no medications. she has smoked one and one half packs of cigarettes daily for 20yrs and drinks alcohol socially. BMI is 31 kg/m2. BP 135/87. Physical exam shows no abnormalities. which of the following contraceptive methods is contraindicated in this patient? A. combination oral contraceptive B. copper IUD C. diaphragm D. progestin IUD E. progestin-only oral contraceptive

A. combination oral contraceptive

A 22-year-old nulligravid woman who takes oral contraceptives comes to your office because she did not have any withdrawal bleeding during her last pill-free week. She has a history of regular menses and has been taking ethinyl estradiol, 35 µg, and norethindrone, 1 mg (Ortho-Novum 1/35, Norinyl 1/35) for almost 1 year, without any problems. A serum pregnancy test is negative 2 weeks after the missed menses, and the patient insists that she takes her pills faithfully as prescribed. She is otherwise healthy and taking no other medications. What would you do now? (A) Perform laboratory evaluation of thyroid function and prolactin and follicle-stimulating hormone (FSH) concentrations. (B) Advise her to stop oral contraceptive use if she plans to conceive in the future because the risk of post-pill amenorrhea is increased. (C) Repeat the pregnancy test because the first test may have been done too early to detect a pregnancy. (D) Continue to prescribe the same oral contraceptive formulation. (E) Perform an ultrasonography to rule out a uterine abnormality.

Answer: D Educational Objective: Properly manage amenorrhea in a patient using combination oral contraceptive agents. The development of amenorrhea while using combination oral contraceptive agents is a relatively frequent occurrence. The amenorrhea that women experience while taking combination pills is secondary to inadequate estrogenic stimulation of the endometrium or to endometrial atrophy caused by the progestational component. In this patient ingesting 35 µg of ethinyl estradiol as part of a "low-dose" pill, it is likely to be secondary to endometrial atrophy. Because of the greater safety of oral contraceptives containing less than 50 µg of estrogen, women should be started on a 35-µg oral contraceptive and, if possible, maintained on such a "low-dose" preparation. If this patient is willing to accept being amenorrheic and does not fail to take her pills, there is no risk in her continuing oral contraceptive use in the normal fashion. The amenorrhea will not adversely affect her health. If she cannot be reassured and wants to have menses, switching to a preparation containing less progestogen (such as Brevicon or Modicon, which contains 0.5 mg of norethindrone, or to Ovcon 35, which contains 0.4 mg of norethindrone) may be effective. Alternatively, the amount of estrogen can be increased by switching to a preparation containing 50 µg of ethinyl estradiol (a so-called "moderate-dose" pill) for three cycles. One might also provide additional estrogen in the form of micronized 17b-estradiol (Estrace, 1 mg), conjugated equine estrogen (Premarin, 0.625 mg), or ethinyl estradiol (Estinyl, 20 µg) daily for the 21 days while taking the oral contraceptive for one to three cycles. Increasing the estrogen amount should rejuvenate the endometrium so that withdrawal bleeding will resume. Other causes of amenorrhea, including hypothyroidism, ovarian failure, various forms of hypothalamic amenorrhea, and hyperprolactinemia, are masked by the use of oral contraceptives. Development of amenorrhea while using these pills is not an indication for endocrine testing. Similarly, there is no evidence that oral contraceptives will lead to so-called post-pill amenorrhea after cessation of use. Women typically resume ovulating within 3 months of discontinuing oral contraceptive use, and 95% to 98% of women begin ovulating within 1 year. It is true that women who had amenorrhea or irregular menses before using oral contraceptive preparations may well return to their former pattern. Other women may develop medical problems while using oral contraceptives that result in amenorrhea only when the pills are discontinued. Women with chronic anovulation can use oral contraceptives, including those women with hypothalamic amenorrhea and polycystic ovarian syndrome. However, these patients should be aware that their anovulation will likely continue after they stop using oral contraceptives. Because this patient's serum pregnancy test is negative and she takes her pills faithfully, she is not pregnant. Commercially available tests for serum human chorionic gonadotropin (hCG) are sufficiently sensitive to detect a pregnancy before a missed menstrual period. By the time of a missed menses, serum hCG levels should be in excess of 100 mIU/mL. It is unlikely that any uterine abnormality is the cause of the patient's amenorrhea. The only abnormality likely to result in amenorrhea in a person with previously regular menses is Asherman syndrome, or intrauterine synechiae. It usually occurs only after an operative procedure, such as dilation and curettage, involving a gravid uterus. It does not occur spontaneously in this setting and has not been associated with amenorrhea related to oral contraceptive use. Thus, ultrasonography is not warranted in this patient.

A patient with benign prostatic hyperplasia presents to the office with increasing difficulty in urination. He has no pain upon urination, but says that "Doc, I can feel my bladder being full, but I just can't urinate". A bladder ultrasound confirms urinary retention. You prescribe a drug and ask him to follow up in 2 weeks. During the follow-up visit, he says that the urinary retention is resolved but that he now feels dizzy and sometimes "blacks out" when he wakes up in the morning. What was the drug that you had prescribed? A Methoscopolamine B Atropine C Prazosin D Finasteride E Furosemide

C Prazosin

A 26-year-old female is using injectable medroxyprogesterone acetate as a method of contraception. Which of the following adverse effects is a concern if she wishes to use this therapy long-term? A. Hyperkalemia. B. Male pattern baldness. C. Osteoporosis. D. Weight loss

C. Osteoporosis. Medroxyprogesterone acetate may contribute to bone loss and predispose patients to osteoporosis and/or fractures. Therefore, the drug should not be continued for more than 2 years if possible. The drug often causes weight gain, not weight loss. The other adverse effects are not associated with medroxyprogesterone.

What are the clinical use of Danazol?

Endometriosis Fibrocystic Breast Hereditary Angioedema

Why is Flutamide administered with Leuprolide?

Flutamide is a non-steroidal antiandrogen that acts as a competitive inhibitor at testosterone receptors. It is used in combination with a GnRH agonist, such as leuprolide, in the treatment of advanced prostate cancer. *The inhibitory effect of flutamide on the tumor's testosterone receptors is beneficial to halt the progression of prostate cancer. However, there is also an unwanted increase in systemic testosterone levels seen with flutamide monotherapy due to reduced negative feedback at the level of the pituitary.* The addition of a leuprolide counters this unwanted increase in testosterone levels by decreasing the release of LH and FSH from the anterior pituitary, thereby reducing androgen production.

MOA of Combined Contraceptives

In the combination pills, the constant level of estrogen supplied continuously suppresses pituitary gonadotropin secretion, thereby removing the stimulus for ovulation. The progesterone in the combination pills serves two functions: first it thickens the cervical mucus secretions, essentially making the vaginal/uterine environment less "receptive" to sperm, and second, it opposes the proliferative effects of estrogen, causing thickening of the uterine lining (which is important in reducing the risk of endometrial cancer from unopposed estrogen)

A 19-year-old primigravid woman at 42 weeks' gestation comes the labor and delivery ward for induction of labor. Her prenatal course was uncomplicated. Examination shows her cervix to be long, thick, closed, and posterior. The fetal heart rate is in the 140s and reactive. The fetus is vertex on ultrasound. Prostaglandin (PGE2) gel is placed intravaginally. One hour later, the patient begins having contractions lasting longer than 2 minutes. The fetal heart rate falls to the 70s. Which of the following is the most appropriate next step in management? A. Administer general anesthesia B. Administer terbutaline C. Perform amnioinfusion D. Start oxytocin E. Perform cesarean delivery

Once patients reach 42 completed weeks of gestation, many physicians will induce labor for post-term pregnancy. This is done to avoid the uncommon but catastrophic outcome of fetal demise and the higher rates of placental insufficiency that develop as patients get further post-term. Prostaglandin (PGE2) gel is an effective agent to use for labor induction. It has been shown to improve the Bishop's score, to shorten the length of labor and delivery, to decrease the amount of oxytocin needed, and to decrease the cesarean delivery rate. The main complication from its use is uterine hyperstimulation. This hyperstimulation is defined as an increased frequency of contractions (greater than 5 every 10 minutes) or an increased length of each contraction (greater than 2 minutes) with evidence of fetal distress. When this hyperstimulation occurs, the patient may be treated with IV or subcutaneous terbutaline. This medication usually has a rapid onset of action in resolving hyperstimulation. IV magnesium sulfate can also be used.

Give three examples of SERMS

Tamoxifen Raloxifen Clomiphene

If both Tamoxifen and Raloxifen are partial agonists of bone, why is only Raloxifen indicated for the treatment of osteoporosis?

Tamoxifen and Raloxifen both are estrogenic on bone tissues (preventing bone loss) and can be used in postmenopausal osteoporosis. However, Tamoxifen is Anti-Estrogenic in premenopausal women (can itself cause osteoporosis) that's why Raloxifen is more famous for this indication.

Selective Estrogen Receptor Modulators (SERMS)

SERMs are a class of estrogen-related compounds that display selective agonism or antagonism for estrogen receptors depending on the tissue type.

How are α1 selective antagonists helpful in the treatment of BPH?

a1 selective antagonists end in -osin Blockade of the A1 A receptors decreases tone in the smooth muscle of the bladder neck and prostate and improves urine flow These agents also cause hypotension as a result ofvascular Alpha 1B receptor blockade Prazosin is selective for A1 A/D receptors found on the prostate and therefore causes the least hypotension


संबंधित स्टडी सेट्स

Texas statutes and rules pertinent to life insurance only questions

View Set

Health Communication HSC4022 Midterm Review

View Set

Ch 4 Comprehension Memory Cognitive Learning

View Set

143 Module 1: FEABS Pharmacology (PRACTICE QUESTIONS)

View Set

Hospitality and Tourism Exam: Quiz 1 (odd numbers )

View Set

L201 Nov. 7 Reading (Contracts Exam)

View Set

Solar System Facts (9 planets)``

View Set

ACCT 292 Chapter 8: Concept Videos

View Set